All Discussions
HitBox
Word
Docs
Hits
pt5
90
97
s3
910
1.3K
14
2.1K
2.8K
Related Threads
LSAT India PT 2 - 2012, S3, Q 10 & 14
Hey Everyone, <br /> <br /> I'm posting here to see if anybody can help me in trying to figure out the answers to two Flaw Questions that appeared …
PT5.S3.Q2 - protect valuable natural resources
I need help with this question<br /> <br /> Admin note: Please review the forum rules.<br /> @"J.Y. Ping" said:<br /> 3. Do not post LSAT questi…
PT5.S3.Q18 - dr. schilling: those who advocate replacing my
Why is (A) right and not (D)? I cannot for the life of me figure this out
PT5.S3.Q12 - the number of north american children
Someone help me out here? I have a vague understanding of why C is the right answer, though I naively selected A. Best explanation I could come up wi…
PT5.S3.Q20 - a certain viral infection
I think I understand why B is right, since middle ear infections are not defined as being bacterial infections in the stimulus. Probably just assumed…
PT5.S3.Q06 - whale's skeleton
I had a hard time understanding why (A) is the right answer. After thinking about it for a day on and off, I came up with this reasoning. Please take…
PT5.S3.Q12 - the number of north american children
Hi All,<br /> <br /> Any chance someone is willing to help me understand why the correct answer choice for this question is C? I cannot seem to fi…
PT5.S3.Q18 - private vs nationalized health insurance
I find myself struggling with Disagree questions. It's difficult to keep the moving parts together and find the overlap. Especially when there are em…
PT5.S3.Q23
For some reason I have MBT question stem, and the correct answer is D.<br /> Can someone explain why D is MBT?<br /> .<br /> Thank you in advance!
PT5.S3.Q01
Hey Guys, anyone able to help me understand this question?<br /> <br /> I interpreted it as: <br /> Terry: WDJ (want decent job) --> C (go to C…
Related Searches
pt5 s3 q15
pt5 s3 q3
pt55 s3 14
pt5 s3 q6
pt5 s3 q1
pt5 s3 q06
pt5 s3 q24
pt92 s3 14
pt5 s3 q23
pt5 s3 q7
Title Search
@title beer
Word 'beer' in title
User Search
@user admin
Only posts by Admin
Text Search
@body beer
Word 'beer' in the text
Phrase Search
"Hello World"
Exact phrase match
Exclude Terms
car -red
Search cars that are not red
"Or" Searches
honda | bmw
'honda' or 'bmw' results
Multi-Field
@(title,body) hello
Both contain 'hello'
WildCard
hond* civ*
Match all with an astrix
Combination
@title bmw @user admin
See more
supported syntax examples
Search Again :: Adv Search
1000 results in 0.002s
<
1
2
3
4
5
6
7
…
34
>
How to negate most and most not?
Now I know the opposite of some is none and the opposite of some .. not is all. What about the opposite of most and most .. not? I can see the opposi…
The Q is PT36,
S3
, #
14
, answer choice C. I think phillip is right by just adding and deleting the "not".
dictronic110
July 2014
Logical Reasoning
PT5
.
S3
.Q12 - the number of north american children
Someone help me out here? I have a vague understanding of why C is the right answer, though I naively selected A. Best explanation I could come up wi…
C is not the right answer for
PT5
.
S3
.Q18, A is.
inactive
February 2017
Logical Reasoning
PT5
.
S3
.Q12 - the number of north american children
Someone help me out here? I have a vague understanding of why C is the right answer, though I naively selected A. Best explanation I could come up wi…
> @"Dillon A. Wright" said:
> C is not the right answer for
PT5
.
S3
.Q18, A is.
Wrong question number. Sorry. I was actually wondering about 12, the obese kids.
extramedium
February 2017
Logical Reasoning
PT5
.
S3
.Q12 - the number of north american children
Someone help me out here? I have a vague understanding of why C is the right answer, though I naively selected A. Best explanation I could come up wi…
Quick correction: This is
PT5
.
S3
.Q12.
I will follow up with a response to your question!
BenjaminSF
February 2017
Logical Reasoning
Does anyone have a strategy sheet for the different types of LR questions?
If anyone could share their strategy sheet for Logical Reasoning question types, it would be greatly appreciated. 7Sage has a really helpful one i…
... . Unproven v. Untrue
PT64
S3
#
14
: “Arnot’s argument is FALSE ... is flawed. Ex: PT3 S2 #
14
: Physicians in training
ntrepanier5
August 2024
Logical Reasoning
LSAT India PT 2 - 2012,
S3
, Q 10 &
14
Hey Everyone, I'm posting here to see if anybody can help me in trying to figure out the answers to two Flaw Questions that appeared in Section 3…
... one would be Q10 in
S3
. I picked A, based off ...
A_Iheduru23
March 2015
General
PT23.
S3
.Q14 - if the proposed tax reduction
Hi guys. I am having trouble with an LR question. PT 23 S3 Q14 Premises: If tax adopted -> Discontinue story hours -> parents inconvenienced …
... LR question.
PT 23
S3
Q14
Premises: If ... /lsat-23-section-3-question-
14
/
Lsatbreakingnews
August 2017
Logical Reasoning
PT43.
S3
.Q01 - photons and neutrinos
http://7sage.com/lsat_explanations/lsat-43-section-3-question-10/ Can anyone help me about the PT43-S3-Q1? I understand A can fill the gap but i…
... help me about the PT43-
S3
-Q1?
I understand ...
JS
September 2013
Logical Reasoning
PT47.
S3
.Q14 - main cause of melanoma
http://7sage.com/lsat_explanations/lsat-47-section-3-question-14/ I'm very confused with this specific question, one of the few LR that I have eve…
... /lsat-47-section-3-question-
14
/
I'm very ...
jschmi25
May 2014
General
PT57.
S3
.Q14 - food labelling regulation
Why does this question trip me up? http://7sage.com/lsat_explanations/lsat-57-section-3-question-14/ I go so caught up in the relatively or seemi…
... /lsat-57-section-3-question-
14
/
I go so ...
Quick Silver
November 2014
Logical Reasoning
PT47.
S3
.Q19 - although high cholesterol levels (wording question)
http://7sage.com/lsat_explanations/lsat-47-section-3-question-19/ When you encounter the word "contributes" on the LSAT, do you take this to be a "c…
... assumed so, but PT 47,
S3
, Q19 threw me off by ...
medb1234
August 2015
Logical Reasoning
PT59.
S3
.Q25 - science writer: all scientists have beliefs
<i>Hey 7Sagers! Here's a question from a student I thought you could help out with:</i> I am looking for someone to critique my attempted negation…
... necessary assumption question Dec 2009,
s3
,q25. Thank you so much ...
inactive
September 2015
Logical Reasoning
Endurance!!
Hey all. I need some endurance tips or something. I'm writing 4 section PTs and after the second section my numbers fall off a table. Today's test: S…
... . LR: 74% S2 RC: 75%
S3
LR: 48% 15 minute break ... endurance.... I dunno. Test is
14
days away.... How do you ...
upcountry77
September 2015
General
PT28
S3
Q04
Regarding *PT28 S3 Q04* The question reads: "Each of the following is supported by the information above except:" Can someone explain why "A" i…
Regarding *PT28
S3
Q04*
The question ...
TheBatman
September 2015
Logical Reasoning
LSAT Prep Test 9 (October 1993) -
S3
- Logic Game 2
LSAT Prep Test 9 (October 1993) - S3 - Logic Game 2 Can someone please help me how to split this game into 4 boards?
LSAT Prep Test 9 (October 1993) -
S3
- Logic Game 2
Can someone please help me how to split this game into 4 boards?
Martin01
October 2015
Logic Games
PT 69, Sec. 1, #
14
We debated this question for a long time in Group BR last night. While almost everyone got the correct answer, we were strongly divided about the act…
... /lsat-69-section-1-question-
14
/
DumbHollywoodActor
November 2015
General
T-
14
Splitters, Extenuating Circumstances
Hi All, Saw a few posts on T-14 chances for splitters. What difference (if any) does URM status make? Same question for an addendum (father was h…
... a few posts on T-
14
chances for splitters. What difference ...
enrique.patino.daly
November 2015
General
Preptest 69 - Section 1 - Question
14
("forest fires")
I had a hard time distinguishing the actual flawed reasoning here. I initially chose A because the logic matched perfectly, but changed my answer to …
... /lsat-69-section-1-question-
14
/
sarkisp23
November 2015
Logical Reasoning
[Ended] Letter of Continuing Interest (LOCI, Waitlist) with David Busis | Thurs. 4/
14
at 9pm ET
<b>If you are on any waitlists, you NEED to attend this webinar!</b> <h1>Letter of Continuing Interest (LOCI) Bootcamp with David Busis</h1> <h4>…
... David Busis
Thursday 4/
14
at 9pm ET
Thu, Apr
14
, 2016 8:00 PM - 8 ...
Nicole Hopkins
March 2016
Webinars
[Ended] Office Hours 6/
14
/16 @ 8:00 PM EST
Come hang out and bring your own questions for another rousing edition of office hours! As always, anything goes--we're here to help you however we c…
... . Please join my meeting, Jun
14
, 2016 at 8:00 PM ...
c.janson35
June 2016
General
PT78.
S3
.Q23 - a spy fails by being caught
Hey everyone, I'm having difficulty determining how the conclusion of the passage is the conclusion. I thought the conclusion was "I was late to m…
... this test (I also got
S3
Q. 17 "identify the conclusion ...
CalPoliSci
September 2016
Logical Reasoning
Blind Review Group | PT 73 | Friday, October
14
| 12 pm EST [Ended]
Hey Gang, BR group is in the daytime. http://opengameart.org/sites/default/files/Daytime_Background_1024x800.png I’ve got no gif game. I’m 4…
... 40.
Friday, October
14
at 12PM ET: PT73
DumbHollywoodActor
October 2016
Study Groups
-
14
PT Curve List
I am planning to supplement my pre-December PTs (76, 77, 78 and 79) with another 6-8 PT's with that have -13 or -14 curves. Basically I'm looking for…
... with that have -13 or -
14
curves. Basically I'm looking ... "Undisclosed" Feb 97 PT (-
14
)
Superprep B (-
14
)
PT 71 ...
Creasey LSAT
November 2016
General
PT67.
S3
.Q04 (G1) - five students
In PT 67 S3 G1 Q4, JY determines that "(A)" is correct for a "which one must be false?" question, and then says, "and you can check the rest; you <b>…
In PT 67
S3
G1 Q4, JY determines that "( ...
steve-10
November 2016
Logic Games
Study Group | PT 72 | Wednesday, December
14
| 12 pm EST [Ended]
Hope to have you there! https://media2.giphy.com/media/aAkNru67Hh40E/200_s.gif <h1>Wednesday, December 14 at 12PM ET: PT72</h1> Click here to …
... .gif
Wednesday, December
14
at 12PM ET: PT72
Wednesday, December
14
, 2016 - PT 72
Wednesday ...
DumbHollywoodActor
December 2016
Study Groups
3.8, 164. Is T-
14
out of reach?
I'm a non-URM and have average softs with 2-3 years of work experience. If I don't mind paying a sticker, which T-14 schools should I apply? Do I ev…
I'm a non-URM and have average softs with 2-3 years of work experience.
If I don't mind paying a sticker, which T-
14
schools should I apply? Do I even have a shot (especially at Michigan, UVA, Northwestern) with such low LSAT?
jaylenosgarage
January 2017
General
JY's List of Live Commentary Video - as of 2/
14
/17
Thank you @jusinhan for you post - JY's response was so great that hopefully it will help us all in the Study Guide section for future reference:) h…
... lsat76.s4
>lsat76.
s3
.game-4
> ... 1-passage
>lsatc2.
s3
>lsatc2.s2< ... -1
>lsat71.
s3
>lsat71.s2. ... lsat68.
s3
>vlsat61.
s3
.game-4
>lsat61.
s3
. ... -passage
>lsat4.
s3
.game-4
> ...
twssmith
February 2017
Study Guides/Cheat Sheets
PT40.
S3
.Q14 - the only clue I have
Got this one right by POE but had a tough time being OK with A. It is definitely something I considered as a flaw in the reasoning. However, the use …
... /lsat-40-section-3-question-
14
/
dcdcdcdcdc
February 2017
Logical Reasoning
[End] FYI: Webinar 9PM EST, March
14
: Kick the Crap Out of Law School by Larry Law Law
Here is promotional material for Larry Law Law's webinar on his course KTCOOLS (Kick the Crap Out of Law School) March Prep for Law School - Befor…
... FREE webinar on Tuesday, March
14
at 9pm Eastern. I'll ... deets:
Time: Mar
14
, 2017 9:00 PM Eastern ...
DumbHollywoodActor
March 2017
General
PT36.
S3
.Q26 - before 1986 physicists
My question here stems from the situation presented in PT36.S3.Q26 (https://7sage.com/lsat_explanations/lsat-36-section-3-question-26/) It seems t…
... the situation presented in PT36.
S3
.Q26 (https://7sage.com/lsat_explanations ...
AnthonyScalia
July 2017
Logical Reasoning
<
1
2
3
4
5
6
7
…
34
>